Compute $lim_{ntoinfty}frac{tfrac{n}{1}+tfrac{n-1}{2}+dots+tfrac{2}{n-1}+tfrac{1}{n}}{ln(n!)}$ [closed]












5












$begingroup$


How can I compute the following limit?
$$
lim_{ntoinfty}frac{dfrac{n}{1}+dfrac{n-1}{2}+dots+dfrac{2}{n-1}+dfrac{1}{n}}{ln(n!)}
$$



I have tried lots of methods, I can't get the answer.
Although I think the limit is $0$, I don't know how to explain it. Please, if someone could help me it would be fantastic.










share|cite|improve this question











$endgroup$



closed as off-topic by RRL, Holo, amWhy, Abcd, Zacky Dec 29 '18 at 12:41


This question appears to be off-topic. The users who voted to close gave this specific reason:


  • "This question is missing context or other details: Please provide additional context, which ideally explains why the question is relevant to you and our community. Some forms of context include: background and motivation, relevant definitions, source, possible strategies, your current progress, why the question is interesting or important, etc." – RRL, Holo, amWhy, Abcd, Zacky

If this question can be reworded to fit the rules in the help center, please edit the question.












  • 3




    $begingroup$
    Can you take a screenshot of what you tried and upload it to us? We like to see what your efforts have been so that we can better help you with this problem.
    $endgroup$
    – Noble Mushtak
    Dec 28 '18 at 21:04






  • 3




    $begingroup$
    For what it's worth, Wolfram Alpha says the sum on the top is $(n+1)H_n-n$. $H_napprox ln n$ for big n, so this gives us $nln n+ln n-n$ over $ln(n!)$. $ln(n!)$ is about $nln n$, so I'm guessing the limit is $1$. However, this is very rough reasoning, so don't trust me on that.
    $endgroup$
    – Noble Mushtak
    Dec 28 '18 at 21:07






  • 2




    $begingroup$
    OK, Wolfram Alpha confirms the limit is $1$.
    $endgroup$
    – Noble Mushtak
    Dec 28 '18 at 21:10










  • $begingroup$
    Hint: Use Stirling's approximation.
    $endgroup$
    – Breakfastisready
    Dec 28 '18 at 21:21










  • $begingroup$
    Thank you everybody!
    $endgroup$
    – M.Martinez
    Dec 29 '18 at 0:28
















5












$begingroup$


How can I compute the following limit?
$$
lim_{ntoinfty}frac{dfrac{n}{1}+dfrac{n-1}{2}+dots+dfrac{2}{n-1}+dfrac{1}{n}}{ln(n!)}
$$



I have tried lots of methods, I can't get the answer.
Although I think the limit is $0$, I don't know how to explain it. Please, if someone could help me it would be fantastic.










share|cite|improve this question











$endgroup$



closed as off-topic by RRL, Holo, amWhy, Abcd, Zacky Dec 29 '18 at 12:41


This question appears to be off-topic. The users who voted to close gave this specific reason:


  • "This question is missing context or other details: Please provide additional context, which ideally explains why the question is relevant to you and our community. Some forms of context include: background and motivation, relevant definitions, source, possible strategies, your current progress, why the question is interesting or important, etc." – RRL, Holo, amWhy, Abcd, Zacky

If this question can be reworded to fit the rules in the help center, please edit the question.












  • 3




    $begingroup$
    Can you take a screenshot of what you tried and upload it to us? We like to see what your efforts have been so that we can better help you with this problem.
    $endgroup$
    – Noble Mushtak
    Dec 28 '18 at 21:04






  • 3




    $begingroup$
    For what it's worth, Wolfram Alpha says the sum on the top is $(n+1)H_n-n$. $H_napprox ln n$ for big n, so this gives us $nln n+ln n-n$ over $ln(n!)$. $ln(n!)$ is about $nln n$, so I'm guessing the limit is $1$. However, this is very rough reasoning, so don't trust me on that.
    $endgroup$
    – Noble Mushtak
    Dec 28 '18 at 21:07






  • 2




    $begingroup$
    OK, Wolfram Alpha confirms the limit is $1$.
    $endgroup$
    – Noble Mushtak
    Dec 28 '18 at 21:10










  • $begingroup$
    Hint: Use Stirling's approximation.
    $endgroup$
    – Breakfastisready
    Dec 28 '18 at 21:21










  • $begingroup$
    Thank you everybody!
    $endgroup$
    – M.Martinez
    Dec 29 '18 at 0:28














5












5








5


1



$begingroup$


How can I compute the following limit?
$$
lim_{ntoinfty}frac{dfrac{n}{1}+dfrac{n-1}{2}+dots+dfrac{2}{n-1}+dfrac{1}{n}}{ln(n!)}
$$



I have tried lots of methods, I can't get the answer.
Although I think the limit is $0$, I don't know how to explain it. Please, if someone could help me it would be fantastic.










share|cite|improve this question











$endgroup$




How can I compute the following limit?
$$
lim_{ntoinfty}frac{dfrac{n}{1}+dfrac{n-1}{2}+dots+dfrac{2}{n-1}+dfrac{1}{n}}{ln(n!)}
$$



I have tried lots of methods, I can't get the answer.
Although I think the limit is $0$, I don't know how to explain it. Please, if someone could help me it would be fantastic.







limits






share|cite|improve this question















share|cite|improve this question













share|cite|improve this question




share|cite|improve this question








edited Dec 28 '18 at 22:15









Euler Pythagoras

58111




58111










asked Dec 28 '18 at 20:57









M.MartinezM.Martinez

343




343




closed as off-topic by RRL, Holo, amWhy, Abcd, Zacky Dec 29 '18 at 12:41


This question appears to be off-topic. The users who voted to close gave this specific reason:


  • "This question is missing context or other details: Please provide additional context, which ideally explains why the question is relevant to you and our community. Some forms of context include: background and motivation, relevant definitions, source, possible strategies, your current progress, why the question is interesting or important, etc." – RRL, Holo, amWhy, Abcd, Zacky

If this question can be reworded to fit the rules in the help center, please edit the question.







closed as off-topic by RRL, Holo, amWhy, Abcd, Zacky Dec 29 '18 at 12:41


This question appears to be off-topic. The users who voted to close gave this specific reason:


  • "This question is missing context or other details: Please provide additional context, which ideally explains why the question is relevant to you and our community. Some forms of context include: background and motivation, relevant definitions, source, possible strategies, your current progress, why the question is interesting or important, etc." – RRL, Holo, amWhy, Abcd, Zacky

If this question can be reworded to fit the rules in the help center, please edit the question.








  • 3




    $begingroup$
    Can you take a screenshot of what you tried and upload it to us? We like to see what your efforts have been so that we can better help you with this problem.
    $endgroup$
    – Noble Mushtak
    Dec 28 '18 at 21:04






  • 3




    $begingroup$
    For what it's worth, Wolfram Alpha says the sum on the top is $(n+1)H_n-n$. $H_napprox ln n$ for big n, so this gives us $nln n+ln n-n$ over $ln(n!)$. $ln(n!)$ is about $nln n$, so I'm guessing the limit is $1$. However, this is very rough reasoning, so don't trust me on that.
    $endgroup$
    – Noble Mushtak
    Dec 28 '18 at 21:07






  • 2




    $begingroup$
    OK, Wolfram Alpha confirms the limit is $1$.
    $endgroup$
    – Noble Mushtak
    Dec 28 '18 at 21:10










  • $begingroup$
    Hint: Use Stirling's approximation.
    $endgroup$
    – Breakfastisready
    Dec 28 '18 at 21:21










  • $begingroup$
    Thank you everybody!
    $endgroup$
    – M.Martinez
    Dec 29 '18 at 0:28














  • 3




    $begingroup$
    Can you take a screenshot of what you tried and upload it to us? We like to see what your efforts have been so that we can better help you with this problem.
    $endgroup$
    – Noble Mushtak
    Dec 28 '18 at 21:04






  • 3




    $begingroup$
    For what it's worth, Wolfram Alpha says the sum on the top is $(n+1)H_n-n$. $H_napprox ln n$ for big n, so this gives us $nln n+ln n-n$ over $ln(n!)$. $ln(n!)$ is about $nln n$, so I'm guessing the limit is $1$. However, this is very rough reasoning, so don't trust me on that.
    $endgroup$
    – Noble Mushtak
    Dec 28 '18 at 21:07






  • 2




    $begingroup$
    OK, Wolfram Alpha confirms the limit is $1$.
    $endgroup$
    – Noble Mushtak
    Dec 28 '18 at 21:10










  • $begingroup$
    Hint: Use Stirling's approximation.
    $endgroup$
    – Breakfastisready
    Dec 28 '18 at 21:21










  • $begingroup$
    Thank you everybody!
    $endgroup$
    – M.Martinez
    Dec 29 '18 at 0:28








3




3




$begingroup$
Can you take a screenshot of what you tried and upload it to us? We like to see what your efforts have been so that we can better help you with this problem.
$endgroup$
– Noble Mushtak
Dec 28 '18 at 21:04




$begingroup$
Can you take a screenshot of what you tried and upload it to us? We like to see what your efforts have been so that we can better help you with this problem.
$endgroup$
– Noble Mushtak
Dec 28 '18 at 21:04




3




3




$begingroup$
For what it's worth, Wolfram Alpha says the sum on the top is $(n+1)H_n-n$. $H_napprox ln n$ for big n, so this gives us $nln n+ln n-n$ over $ln(n!)$. $ln(n!)$ is about $nln n$, so I'm guessing the limit is $1$. However, this is very rough reasoning, so don't trust me on that.
$endgroup$
– Noble Mushtak
Dec 28 '18 at 21:07




$begingroup$
For what it's worth, Wolfram Alpha says the sum on the top is $(n+1)H_n-n$. $H_napprox ln n$ for big n, so this gives us $nln n+ln n-n$ over $ln(n!)$. $ln(n!)$ is about $nln n$, so I'm guessing the limit is $1$. However, this is very rough reasoning, so don't trust me on that.
$endgroup$
– Noble Mushtak
Dec 28 '18 at 21:07




2




2




$begingroup$
OK, Wolfram Alpha confirms the limit is $1$.
$endgroup$
– Noble Mushtak
Dec 28 '18 at 21:10




$begingroup$
OK, Wolfram Alpha confirms the limit is $1$.
$endgroup$
– Noble Mushtak
Dec 28 '18 at 21:10












$begingroup$
Hint: Use Stirling's approximation.
$endgroup$
– Breakfastisready
Dec 28 '18 at 21:21




$begingroup$
Hint: Use Stirling's approximation.
$endgroup$
– Breakfastisready
Dec 28 '18 at 21:21












$begingroup$
Thank you everybody!
$endgroup$
– M.Martinez
Dec 29 '18 at 0:28




$begingroup$
Thank you everybody!
$endgroup$
– M.Martinez
Dec 29 '18 at 0:28










2 Answers
2






active

oldest

votes


















5












$begingroup$

Let $H_n:= sumlimits_{k=1}^n frac{1}{k}$.



$ln(n!)sim n(ln(n)-1)$ by Stirling's formula.



As for the numerator, it is $nH_n-frac{1}{2}-frac{2}{3}-cdots-frac{n-1}{n}= nH_n-(n-1)+H_n-1$.



There are very strong estimations for $H_n$, for example $H_n= ln(n)+gamma+O(frac{1}{n})$.



Putting all this together yields that the limit is $1$, you can even obtain a nice error term that the sequence is in fact $1+O(frac{1}{ln(n)})$.






share|cite|improve this answer









$endgroup$













  • $begingroup$
    Thank you fo helping!!!
    $endgroup$
    – M.Martinez
    Dec 29 '18 at 0:28






  • 1




    $begingroup$
    I am glad I could help. If you think this answers your question, you should accept the answer.
    $endgroup$
    – A. Pongrácz
    Dec 29 '18 at 7:45



















3












$begingroup$

We can apply Stolz-Cesaro
taking $a_n=(n+1)H_n-n$ and $b_n=ln(n!)$:
$$l=lim_{nto infty} frac{a_{n+1}-a_n}{b_{n+1}-b_n}=lim_{ntoinfty} frac{(n+2)H_{n+1}-(n+1)-(n+1)H_n +n}{ln((n+1)!)-ln(n!)}=lim_{nto infty} frac{H_{n+1}}{ln(n+1)}$$
And since $H_n approx ln n+gamma +Oleft(frac1nright)$ we easily get:
$$l=lim_{nto infty} frac{a_{n+1}-a_n}{b_{n+1}-b_n}=1Rightarrow lim_{ntoinfty} frac{a_n}{b_n}=1$$






share|cite|improve this answer











$endgroup$









  • 1




    $begingroup$
    You have a few typos to fix, like $frac{(n+2)H_{n+1}...}{...}$
    $endgroup$
    – rtybase
    Dec 28 '18 at 22:03






  • 1




    $begingroup$
    One more, after all the simplifications, you should have $limlimits_{nrightarrowinfty}frac{H_{n+1}}{ln{(n+1)}}$
    $endgroup$
    – rtybase
    Dec 28 '18 at 22:11






  • 1




    $begingroup$
    Thank you for correcting me!
    $endgroup$
    – Zacky
    Dec 28 '18 at 22:14


















2 Answers
2






active

oldest

votes








2 Answers
2






active

oldest

votes









active

oldest

votes






active

oldest

votes









5












$begingroup$

Let $H_n:= sumlimits_{k=1}^n frac{1}{k}$.



$ln(n!)sim n(ln(n)-1)$ by Stirling's formula.



As for the numerator, it is $nH_n-frac{1}{2}-frac{2}{3}-cdots-frac{n-1}{n}= nH_n-(n-1)+H_n-1$.



There are very strong estimations for $H_n$, for example $H_n= ln(n)+gamma+O(frac{1}{n})$.



Putting all this together yields that the limit is $1$, you can even obtain a nice error term that the sequence is in fact $1+O(frac{1}{ln(n)})$.






share|cite|improve this answer









$endgroup$













  • $begingroup$
    Thank you fo helping!!!
    $endgroup$
    – M.Martinez
    Dec 29 '18 at 0:28






  • 1




    $begingroup$
    I am glad I could help. If you think this answers your question, you should accept the answer.
    $endgroup$
    – A. Pongrácz
    Dec 29 '18 at 7:45
















5












$begingroup$

Let $H_n:= sumlimits_{k=1}^n frac{1}{k}$.



$ln(n!)sim n(ln(n)-1)$ by Stirling's formula.



As for the numerator, it is $nH_n-frac{1}{2}-frac{2}{3}-cdots-frac{n-1}{n}= nH_n-(n-1)+H_n-1$.



There are very strong estimations for $H_n$, for example $H_n= ln(n)+gamma+O(frac{1}{n})$.



Putting all this together yields that the limit is $1$, you can even obtain a nice error term that the sequence is in fact $1+O(frac{1}{ln(n)})$.






share|cite|improve this answer









$endgroup$













  • $begingroup$
    Thank you fo helping!!!
    $endgroup$
    – M.Martinez
    Dec 29 '18 at 0:28






  • 1




    $begingroup$
    I am glad I could help. If you think this answers your question, you should accept the answer.
    $endgroup$
    – A. Pongrácz
    Dec 29 '18 at 7:45














5












5








5





$begingroup$

Let $H_n:= sumlimits_{k=1}^n frac{1}{k}$.



$ln(n!)sim n(ln(n)-1)$ by Stirling's formula.



As for the numerator, it is $nH_n-frac{1}{2}-frac{2}{3}-cdots-frac{n-1}{n}= nH_n-(n-1)+H_n-1$.



There are very strong estimations for $H_n$, for example $H_n= ln(n)+gamma+O(frac{1}{n})$.



Putting all this together yields that the limit is $1$, you can even obtain a nice error term that the sequence is in fact $1+O(frac{1}{ln(n)})$.






share|cite|improve this answer









$endgroup$



Let $H_n:= sumlimits_{k=1}^n frac{1}{k}$.



$ln(n!)sim n(ln(n)-1)$ by Stirling's formula.



As for the numerator, it is $nH_n-frac{1}{2}-frac{2}{3}-cdots-frac{n-1}{n}= nH_n-(n-1)+H_n-1$.



There are very strong estimations for $H_n$, for example $H_n= ln(n)+gamma+O(frac{1}{n})$.



Putting all this together yields that the limit is $1$, you can even obtain a nice error term that the sequence is in fact $1+O(frac{1}{ln(n)})$.







share|cite|improve this answer












share|cite|improve this answer



share|cite|improve this answer










answered Dec 28 '18 at 21:41









A. PongráczA. Pongrácz

5,9531929




5,9531929












  • $begingroup$
    Thank you fo helping!!!
    $endgroup$
    – M.Martinez
    Dec 29 '18 at 0:28






  • 1




    $begingroup$
    I am glad I could help. If you think this answers your question, you should accept the answer.
    $endgroup$
    – A. Pongrácz
    Dec 29 '18 at 7:45


















  • $begingroup$
    Thank you fo helping!!!
    $endgroup$
    – M.Martinez
    Dec 29 '18 at 0:28






  • 1




    $begingroup$
    I am glad I could help. If you think this answers your question, you should accept the answer.
    $endgroup$
    – A. Pongrácz
    Dec 29 '18 at 7:45
















$begingroup$
Thank you fo helping!!!
$endgroup$
– M.Martinez
Dec 29 '18 at 0:28




$begingroup$
Thank you fo helping!!!
$endgroup$
– M.Martinez
Dec 29 '18 at 0:28




1




1




$begingroup$
I am glad I could help. If you think this answers your question, you should accept the answer.
$endgroup$
– A. Pongrácz
Dec 29 '18 at 7:45




$begingroup$
I am glad I could help. If you think this answers your question, you should accept the answer.
$endgroup$
– A. Pongrácz
Dec 29 '18 at 7:45











3












$begingroup$

We can apply Stolz-Cesaro
taking $a_n=(n+1)H_n-n$ and $b_n=ln(n!)$:
$$l=lim_{nto infty} frac{a_{n+1}-a_n}{b_{n+1}-b_n}=lim_{ntoinfty} frac{(n+2)H_{n+1}-(n+1)-(n+1)H_n +n}{ln((n+1)!)-ln(n!)}=lim_{nto infty} frac{H_{n+1}}{ln(n+1)}$$
And since $H_n approx ln n+gamma +Oleft(frac1nright)$ we easily get:
$$l=lim_{nto infty} frac{a_{n+1}-a_n}{b_{n+1}-b_n}=1Rightarrow lim_{ntoinfty} frac{a_n}{b_n}=1$$






share|cite|improve this answer











$endgroup$









  • 1




    $begingroup$
    You have a few typos to fix, like $frac{(n+2)H_{n+1}...}{...}$
    $endgroup$
    – rtybase
    Dec 28 '18 at 22:03






  • 1




    $begingroup$
    One more, after all the simplifications, you should have $limlimits_{nrightarrowinfty}frac{H_{n+1}}{ln{(n+1)}}$
    $endgroup$
    – rtybase
    Dec 28 '18 at 22:11






  • 1




    $begingroup$
    Thank you for correcting me!
    $endgroup$
    – Zacky
    Dec 28 '18 at 22:14
















3












$begingroup$

We can apply Stolz-Cesaro
taking $a_n=(n+1)H_n-n$ and $b_n=ln(n!)$:
$$l=lim_{nto infty} frac{a_{n+1}-a_n}{b_{n+1}-b_n}=lim_{ntoinfty} frac{(n+2)H_{n+1}-(n+1)-(n+1)H_n +n}{ln((n+1)!)-ln(n!)}=lim_{nto infty} frac{H_{n+1}}{ln(n+1)}$$
And since $H_n approx ln n+gamma +Oleft(frac1nright)$ we easily get:
$$l=lim_{nto infty} frac{a_{n+1}-a_n}{b_{n+1}-b_n}=1Rightarrow lim_{ntoinfty} frac{a_n}{b_n}=1$$






share|cite|improve this answer











$endgroup$









  • 1




    $begingroup$
    You have a few typos to fix, like $frac{(n+2)H_{n+1}...}{...}$
    $endgroup$
    – rtybase
    Dec 28 '18 at 22:03






  • 1




    $begingroup$
    One more, after all the simplifications, you should have $limlimits_{nrightarrowinfty}frac{H_{n+1}}{ln{(n+1)}}$
    $endgroup$
    – rtybase
    Dec 28 '18 at 22:11






  • 1




    $begingroup$
    Thank you for correcting me!
    $endgroup$
    – Zacky
    Dec 28 '18 at 22:14














3












3








3





$begingroup$

We can apply Stolz-Cesaro
taking $a_n=(n+1)H_n-n$ and $b_n=ln(n!)$:
$$l=lim_{nto infty} frac{a_{n+1}-a_n}{b_{n+1}-b_n}=lim_{ntoinfty} frac{(n+2)H_{n+1}-(n+1)-(n+1)H_n +n}{ln((n+1)!)-ln(n!)}=lim_{nto infty} frac{H_{n+1}}{ln(n+1)}$$
And since $H_n approx ln n+gamma +Oleft(frac1nright)$ we easily get:
$$l=lim_{nto infty} frac{a_{n+1}-a_n}{b_{n+1}-b_n}=1Rightarrow lim_{ntoinfty} frac{a_n}{b_n}=1$$






share|cite|improve this answer











$endgroup$



We can apply Stolz-Cesaro
taking $a_n=(n+1)H_n-n$ and $b_n=ln(n!)$:
$$l=lim_{nto infty} frac{a_{n+1}-a_n}{b_{n+1}-b_n}=lim_{ntoinfty} frac{(n+2)H_{n+1}-(n+1)-(n+1)H_n +n}{ln((n+1)!)-ln(n!)}=lim_{nto infty} frac{H_{n+1}}{ln(n+1)}$$
And since $H_n approx ln n+gamma +Oleft(frac1nright)$ we easily get:
$$l=lim_{nto infty} frac{a_{n+1}-a_n}{b_{n+1}-b_n}=1Rightarrow lim_{ntoinfty} frac{a_n}{b_n}=1$$







share|cite|improve this answer














share|cite|improve this answer



share|cite|improve this answer








edited Dec 28 '18 at 22:13

























answered Dec 28 '18 at 21:56









ZackyZacky

7,1501961




7,1501961








  • 1




    $begingroup$
    You have a few typos to fix, like $frac{(n+2)H_{n+1}...}{...}$
    $endgroup$
    – rtybase
    Dec 28 '18 at 22:03






  • 1




    $begingroup$
    One more, after all the simplifications, you should have $limlimits_{nrightarrowinfty}frac{H_{n+1}}{ln{(n+1)}}$
    $endgroup$
    – rtybase
    Dec 28 '18 at 22:11






  • 1




    $begingroup$
    Thank you for correcting me!
    $endgroup$
    – Zacky
    Dec 28 '18 at 22:14














  • 1




    $begingroup$
    You have a few typos to fix, like $frac{(n+2)H_{n+1}...}{...}$
    $endgroup$
    – rtybase
    Dec 28 '18 at 22:03






  • 1




    $begingroup$
    One more, after all the simplifications, you should have $limlimits_{nrightarrowinfty}frac{H_{n+1}}{ln{(n+1)}}$
    $endgroup$
    – rtybase
    Dec 28 '18 at 22:11






  • 1




    $begingroup$
    Thank you for correcting me!
    $endgroup$
    – Zacky
    Dec 28 '18 at 22:14








1




1




$begingroup$
You have a few typos to fix, like $frac{(n+2)H_{n+1}...}{...}$
$endgroup$
– rtybase
Dec 28 '18 at 22:03




$begingroup$
You have a few typos to fix, like $frac{(n+2)H_{n+1}...}{...}$
$endgroup$
– rtybase
Dec 28 '18 at 22:03




1




1




$begingroup$
One more, after all the simplifications, you should have $limlimits_{nrightarrowinfty}frac{H_{n+1}}{ln{(n+1)}}$
$endgroup$
– rtybase
Dec 28 '18 at 22:11




$begingroup$
One more, after all the simplifications, you should have $limlimits_{nrightarrowinfty}frac{H_{n+1}}{ln{(n+1)}}$
$endgroup$
– rtybase
Dec 28 '18 at 22:11




1




1




$begingroup$
Thank you for correcting me!
$endgroup$
– Zacky
Dec 28 '18 at 22:14




$begingroup$
Thank you for correcting me!
$endgroup$
– Zacky
Dec 28 '18 at 22:14



Popular posts from this blog

Quarter-circle Tiles

build a pushdown automaton that recognizes the reverse language of a given pushdown automaton?

Mont Emei